0 Daumen
128 Aufrufe

Hallo,

Ich habe mich erst gefragt, wie man beim Konvergenzbeweis für Folgen vorgehen kann. Bei Reihen weiß ich bereits, dass man hier unterschiedliche Konvergenzkriterien (z.B. Wurzelkriterium) anwenden kann. Bei den Folgen bin ich mir allerdings etwas unsicher, wie man dies beweisen kann. Gilt es hier zuerst den Grenzwert mit Limes zu bestimmen oder evtl die Monotonie anzugeben? Es ist mir bewusst, dass dies je nach Folge unterschiedlich ist, aber vielleicht gibt es eine Art „Kochrezept“.

LG

Avatar von

1 Antwort

0 Daumen
 
Beste Antwort

Hallo :-)

Was zu zeigen ist, ist in den Definitionen/Sätzen/Lemmas usw. verankert und gibt ja schon so den Leitfaden, was zu machen ist. Die Kunst ist dann eher, wie man das umsetzt.

Eine übliche Definition zur Konvergenz reeller Folgen kann man so formulieren:

Eine rellee Folge \((a_n)_{n\in \N}\subseteq \R\) konvergiert gegen \(a\in \R\), falls gilt:

$$ \forall \varepsilon>0 \ \exists N_{\varepsilon}\in \N \ \forall n\in \N_{\geq N_{\varepsilon}}: |a_n-a|<\varepsilon$$


Finde also zu jedem \(\varepsilon>0\) eine natürliche Zahl (Index) \(N_{\varepsilon}\in \N\), sodass ab diesem Index (\(\forall n\in \N_{\geq N_{\varepsilon}}\)) der Fehler \(|a_n-a|\) den Wert \(\varepsilon>0\) unterschreitet.

Um so ein explizites \(N_{\varepsilon}\in \N\) zu finden hilft es oft, den Fehler \(|a_n-a|\) nachoben abzuschätzen, um so einfachere Ungleichungen mit \(...<\varepsilon\) zu erhalten, die man dann nach einem \(N_{\varepsilon}\in \N\) auflöst.


Hier mal ein Beispiel:

Sei \((a_n)_{n\in \N_{\geq 1}}\) mit \(a_n:=\frac{\sqrt{n^2+4}}{2n}\).

Zeige \(\lim\limits_{n\to \infty} a_n=\frac{1}{2}\).

Betrachte also zunächst den Fehler

$$ |a_n-a|=\left|\frac{\sqrt{n^2+4}}{2n}-\frac{1}{2} \right|=\left|\frac{\sqrt{n^2+4}}{2n}-\frac{n}{2n} \right|=\left|\frac{\sqrt{n^2+4}-n}{2n} \right|\\[15pt]=\left|\frac{(\sqrt{n^2+4}-n)(\sqrt{n^2+4}+n)}{2n(\sqrt{n^2+4}+n)} \right|=\left|\frac{n^2+4-n^2}{2n(\sqrt{n^2+4}+n)} \right|=\left|\frac{4}{2n(\sqrt{n^2+4}+n)} \right|\\[15pt]= \left|\frac{2}{n(\sqrt{n^2+4}+n)} \right|$$

ab hier kann man jetzt verschiedenartig abschätzen:

1.) $$\left|\frac{2}{n(\sqrt{n^2+4}+n)} \right|\stackrel{n\in \N_{\geq 1}}{\leq} \left|\frac{2n}{n(\sqrt{n^2+4}+n)} \right|=\left|\frac{2}{\sqrt{n^2+4}+n} \right|\leq \left|\frac{2}{n} \right|=\frac{2}{n} \stackrel{n\geq N_{\varepsilon}}{\leq}\frac{2}{N_{\varepsilon}}\stackrel{!}{<} \varepsilon$$

Wähle hier also \(N_{\varepsilon}\in \N_{\geq 1}\) durch \(N_{\varepsilon}>\frac{2}{\varepsilon}\).


2.) $$\left|\frac{2}{n(\sqrt{n^2+4}+n)} \right|\leq \left|\frac{2}{n\cdot n}\right|=\frac{2}{n^2}\stackrel{n\geq N_{\varepsilon}}{\leq}\frac{2}{N_{\varepsilon}^2}\stackrel{!}{<} \varepsilon $$

Wähle hier also \(N_{\varepsilon}\in \N_{\geq 1}\) durch \(N_{\varepsilon}>\sqrt{\frac{2}{\varepsilon}}\).


Und so kann man das jetzt in gefühlt endlos vielen Varianten machen. Du musst dich halt für eine (möglichst einfache) Variante entscheiden. Hast du nun dein \(N_{\varepsilon}\in \N_{\geq 1}\) gefunden, dann schreibst du alles nochmal zusammenfassend auf, was dann den üblichen Konvergenzbweis darstellt, da es in einem Beweis zunächst uninteressant ist, wie man auf eine bestimmte Kenngröße gekommen ist. Man präsentiert also den Gedankengang so, dass dies mit der Definition konform ist: Ich nehme mal Version 1.)

Sei \(\varepsilon>0\) beliebig und wähle \(N_{\varepsilon}\in \N_{\geq 1}\) durch \(N_{\varepsilon}>\frac{2}{\varepsilon}\). Dann gilt für alle \(n\in \N_{\geq N_{\varepsilon}}\)

$$ |a_n-a|=\left|\frac{\sqrt{n^2+4}}{2n}-\frac{1}{2} \right|=\left|\frac{\sqrt{n^2+4}}{2n}-\frac{n}{2n} \right|=\left|\frac{\sqrt{n^2+4}-n}{2n} \right|\\[15pt]=\left|\frac{(\sqrt{n^2+4}-n)(\sqrt{n^2+4}+n)}{2n(\sqrt{n^2+4}+n)} \right|=\left|\frac{n^2+4-n^2}{2n(\sqrt{n^2+4}+n)} \right|=\left|\frac{4}{2n(\sqrt{n^2+4}+n)} \right|\\[15pt]= \left|\frac{2}{n(\sqrt{n^2+4}+n)} \right|\stackrel{n\in \N_{\geq 1}}{\leq} \left|\frac{2n}{n(\sqrt{n^2+4}+n)} \right|=\left|\frac{2}{\sqrt{n^2+4}+n} \right|\leq \left|\frac{2}{n} \right|=\frac{2}{n} \stackrel{n\geq N_{\varepsilon}}{\leq}\frac{2}{N_{\varepsilon}}< \varepsilon$$

Damit ist \(a_n\) gegen \(\frac{1}{2}\) konvergent.

Avatar von 14 k

Ein anderes Problem?

Stell deine Frage

Willkommen bei der Mathelounge! Stell deine Frage einfach und kostenlos

x
Made by a lovely community